LSAT and Law School Admissions Forum

Get expert LSAT preparation and law school admissions advice from PowerScore Test Preparation.

User avatar
 Dave Killoran
PowerScore Staff
  • PowerScore Staff
  • Posts: 5862
  • Joined: Mar 25, 2011
|
#88206
Complete Question Explanation
(The complete setup for this game can be found here: lsat/viewtopic.php?f=172&p=88204#p88204)

The correct answer choice is (A)

In a Not Necessarily True question, the correct answer is Not Necessarily True, and the four incorrect answers Must Be True.

If R is lowest in cost, then from the last rule S must be fifth. From the first rule, K must then be the accepted bid, and must be second or third. From the second rule, H must be lower cost in than K. Thus, H must be second and K must be third:

G4-Q23-d1.png

The only remaining uncertainty involves J and T. As either configuration of J and T satisfies all of the rules, their exact placement cannot be determined:

G4-Q23-d2.png

Because this is a Not Necessarily True question, and J and T are the only variables that are uncertain, you should look for J or T in the answer choices. Answer choice (A) is the only answer to contain either, and because J does not have to be highest in cost, answer choice (A) is correct. Each of the other four answer choices must be true according to the diagram above, and thus each is incorrect.
You do not have the required permissions to view the files attached to this post.

Get the most out of your LSAT Prep Plus subscription.

Analyze and track your performance with our Testing and Analytics Package.